LSAT and Law School Admissions Forum

Get expert LSAT preparation and law school admissions advice from PowerScore Test Preparation.

 Administrator
PowerScore Staff
  • PowerScore Staff
  • Posts: 8917
  • Joined: Feb 02, 2011
|
#73670
Complete Question Explanation

Assumption. The correct answer choice is (B).

The conclusion of the argument is that it is important to exercise the muscles on each side of the spine equally if one is to maintain a healthy back. The evidence is that balanced muscles are required for a healthy back. Students should take note of a gap in the argument, by which we mean that there is something mentioned in the premises that is not in the conclusion, and something mentioned in the conclusion that is not mentioned in the premises. Here, the premises and conclusion both talk about a healthy back, so there is no gap there, but the premises talk about "balanced muscles" while the conclusion talks about exercising both sides equally. The gap, then, is that equal exercise on both sides is required in order to have balanced muscles. Look for the answer that connects these two "rogue" concepts, building a bridge from the premises to the conclusion.

Answer choice (A): This answer alludes to balanced muscles, but does not connect that idea to the concept of equal exercise.

Answer choice (B): This is the correct answer choice. This answer appears to connect our two rogue elements, although it does so in the contrapositive - unequal exercise leads to unbalanced muscles, rather than balanced muscles require equal exercise. The effect, however, is the same.

Answer choice (C): This answer is something like a Mistaken Reversal of the conclusion, and so is incorrect. Also note that it fails to address the issue of balanced muscles, but only talks about the relationship between equal exercise and a healthy back. No bridge is built here between our "rogue" elements, and so this answer is a loser.

Answer choice (D): While this answer bears some resemblance to the correct answer, it goes too far in saying that one's back will be damaged when all the author had to assume was that the back muscles would be unbalanced. Beware of extreme, exaggerated answers to Assumption questions.

Answer choice (E): No assumptions are required about daily exercise. The argument is about equal exercise, however often or infrequent that might be.
 jmaclan01
  • Posts: 5
  • Joined: Dec 13, 2013
|
#13324
I'm not sure if I broke this argument down correctly.

MHB :arrow: EMOE
MHB = Maintain healthy back
EMOE = Exercise muscles on opposite sides equally

I believe this is the conclusion while the second sentence is a premise. Does the second sentence contain a conditional statement as well? Also, is there a gap in assuming "exercising the muscles on opposite sides of the spine equally" is equivalent to "balanced muscle development"? I thought the assumption question might attack that gap.
 Robert Carroll
PowerScore Staff
  • PowerScore Staff
  • Posts: 1787
  • Joined: Dec 06, 2013
|
#13330
j,

This is an excellent start. The first sentence is the conclusion and you've identified the conditional form of it. You're also right that the second sentence is a conditional, with "balanced muscle development" being the necessary condition (because it says this is "needed") and "maintain a healthy back" the sufficient condition. Let "BMD" be an abbreviation for that necessary condition, and keeping the other abbreviations that you used in your post:

MHB :arrow: BMD

Because the conditional in the premise connects MHB and BMD, but the conclusion connects MHB and EMOE, it's natural to look for an assumption that connects EMOE to BMD. For instance, if the following were assumed:

BMD :arrow: EMOE

then, because there is a chain of conditionals, we could combine this with the second sentence to get what the conclusion says.

Look for the answer choice that connects those!

I think this discussion answers your question about the gap - "balanced muscle development" and "exercising muscles on opposite sides equally" have not yet been connected, and the correct answer choice in this question will connect them. That's because the assumption in this question is a Supporter Assumption - it closes the gap by linking elements together.

You were definitely on the right track with this one, and I hope this discussion allows you to understand what the right answer choice is.

Robert
 jmaclan01
  • Posts: 5
  • Joined: Dec 13, 2013
|
#13336
I see the correct answer is linking the two but as a contrapositive. I appreciate your help!
 moalkhaf
  • Posts: 10
  • Joined: Aug 30, 2017
|
#40882
Good morning. I was wondering if someone could look at this practice question from the 2007 LSAT and tell me if I used the assumption negation technique correctly. Not that the capitalized word is the negation.

When exercising the muscles in one’s back, it is
important, in order to maintain a healthy back, to
exercise the muscles on opposite sides of the spine
equally. After all, balanced muscle development is
needed to maintain a healthy back, since the muscles on
opposite sides of the spine must pull equally in
opposing directions to keep the back in proper
alignment and protect the spine.

Which one of the following is an assumption required
by the argument?

(A) Muscles on opposite sides of the spine that are
equally well developed will NOT be enough to keep
the back in proper alignment.

(B) Exercising the muscles on opposite sides of the
spine unequally DOES NOT tend to lead to unbalanced
muscle development.

(C) Provided that one exercises the muscles on
opposite sides of the spine equally, one will NOT
have a generally healthy back.

(D) If the muscles on opposite sides of the spine are NOT
exercised unequally, one’s back will be
irreparably damaged.

(E) One should NOT exercise daily to ensure that the
muscles on opposite sides of the spine keep
the back in proper alignment

Why is C wrong. It directly contradicts what the author is arguing by claiming that excersizing the muscles on opposite sides of the spine equally will not lead to a healthy back
 Francis O'Rourke
PowerScore Staff
  • PowerScore Staff
  • Posts: 471
  • Joined: Mar 10, 2017
|
#40993
Hi Moalkhaf,

The speaker's conclusion was that if you want to maintain a healthy back, you must equally exercise the muscles on either side of the spine.

The evidence given for this argument was that balanced muscle development is needed to maintain a healthy back.

Conclusion: MHB :arrow: EMOE
Premise: ... MHB :arrow: BMD

As Robert Carroll wrote above
Because the conditional in the premise connects MHB and BMD, but the conclusion connects MHB and EMOE, it's natural to look for an assumption that connects EMOE to BMD. For instance, if the following were assumed:

BMD :arrow: EMOE

then, because there is a chain of conditionals, we could combine this with the second sentence to get what the conclusion says.

Look for the answer choice that connects those!
Answer choice (B) connects the information perfectly! it states that NOT exercising muscles on either side equally usually leads to not balanced muscle development. Since the author is apparently assuming that balanced muscle development requires exercising muscles on either side equally, the argument must assume that not exercising muscles equally can lead to unbalanced muscles. If the author did not believe this, then there would be no reason to conclude that we need to exercise both sides equally.


Answer choice (C) tells us that a guaranteed way to maintain a healthy back is to exercise muscles on either side of the spine equally. If you want to diagram this statement, you can do so this way:

..... Exercise muscles on opposite sides equally :arrow: generally healthy back

The author did not attempt to make an argument about guaranteeing a healthy back, but what is necessary for a healthy back, so this statement is no necessary for the argument.

It is possible for the speaker to believe the logical negation of this answer choice and still make the argument. The logical negation of this answer choice is that "exercising muscles on either side does not necessarily guarantee a generally healthy back". Since the argument concerns what is required for a healthy back, it is okay if exercising muscles on either side does not always guarantee a healthy back.

It looks like the logical negation that you provided was too extreme. Make sure to remember that when you negate a conditional statement, you only negating the necessity of the relationship. So, the logical negation of ..... if A, then B ..... is ..... If A, then not necessarily B.
 blade21cn
  • Posts: 100
  • Joined: May 21, 2019
|
#73702
According to the admin's explanation, (B) bridges the gap "Balanced muscle development → Exercise the opposite muscles equally" by way of its contrapositive. But (B) states "Exercising the opposite muscles unequally TENDS TO lead to unbalanced muscle development," which is a "most" statement. I thought "most" statements do not have contrapositives. So I'm reluctant to use formal logic here and transform (B) the way the admin did. Any thoughts?
User avatar
 KelseyWoods
PowerScore Staff
  • PowerScore Staff
  • Posts: 1079
  • Joined: Jun 26, 2013
|
#73713
Hi Blade21cn!

You are correct that "most" statements do not have contrapositives and you could interpret answer choice (B) as a "most" statement. But I wouldn't overthink this question with a formal logic angle. Answer choice (B) is the best answer and if you negate it, it attacks the argument.

Francis had a good description for why answer choice (B) is the best above:
Answer choice (B) connects the information perfectly! it states that NOT exercising muscles on either side equally usually leads to not balanced muscle development. Since the author is apparently assuming that balanced muscle development requires exercising muscles on either side equally, the argument must assume that not exercising muscles equally can lead to unbalanced muscles. If the author did not believe this, then there would be no reason to conclude that we need to exercise both sides equally.
Hope this helps!

Best,
Kelsey
User avatar
 Henry Z
  • Posts: 61
  • Joined: Apr 16, 2022
|
#96734
This question is challenging for me because it has so many conditionals. I read the thread and found no one mentioned the last part of the stimulus ("since..."): proper alignment and protect the spine -->exercise muscles on opposite sides equally.

My anticipation of the answer was: balanced muscle development -->proper alignment and protect the spine. I think it's a legit assumption but it's not what the correct answer looks like. The whole question just ignores the last conditional.

So my question is, under the time pressure, how do we know which part is relevant, which part is not? How can I know I can just stop at "since"? Is it because "since" suggests a causation, not conditional, so it can't fit in the logical chain of the stimulus?
 Robert Carroll
PowerScore Staff
  • PowerScore Staff
  • Posts: 1787
  • Joined: Dec 06, 2013
|
#97416
Henry Z,

That last conditional can certainly be diagrammed. It is a premise, as indicated by the premise indicator "since". The issue with your prephrase is that I don't see how this is a necessary assumption of the argument. Focus on the conclusion, as always - the conclusion mentions exercising on opposite sides equally, something never discussed by the premises.

Robert Carroll

Get the most out of your LSAT Prep Plus subscription.

Analyze and track your performance with our Testing and Analytics Package.